Need help with expected value of probability distribution

Click For Summary
The discussion focuses on calculating the expected value of a probability distribution using the integral formula involving a Gaussian function. The user is attempting various substitutions but struggles to simplify the integral effectively. They suspect the expected value might be related to the term "dt" but are confused about the contributions from different parts of the integral. A hint is provided that the integrand is an odd function, which could simplify the calculation. The user expresses gratitude for the assistance while still feeling lost on the solution.
andresthor
Messages
3
Reaction score
0

Homework Statement



I'm trying to find the expected value of a probability distribution.

Homework Equations



\int_{-\infty}^\infty xP(x,t) = \int_{-\infty}^\infty x \frac{1}{\sqrt{4\pi Dt}}e^{-\frac{(x-dt)^2}{4Dt}}dx

The Attempt at a Solution



I expect the value to be something like $dt$ but then again I might be way off.
I've tried some different substitutions but have had no luck. One example of what I've been trying:

<br /> \begin{equation}<br /> \frac{1}{\sqrt{4\pi Dt}}\int_{-\infty}^\infty x e^{-\frac{(x-dt)^2}{4Dt}}dx=\left/ u=x-dt\right/ = \frac{1}{\sqrt{4\pi Dt}}\int_{-\infty}^\infty (u+dt) e^{-\frac{u^2}{4Dt}}du = \frac{1}{\sqrt{4\pi Dt}}\left(\int_{-\infty}^\infty u e^{-\frac{u^2}{4Dt}}du + dt \int_{-\infty}^\infty e^{-\frac{u^2}{4Dt}}du\right)<br /> \end{equation}<br />

But then I get nowhere.
 
Physics news on Phys.org
For an integral of the form

\int xe^{-k(x-a)^2} dx

I would write it as

\int (x-a)e^{-k(x-a)^2} dx+ a\int e^{-k(x-a)^2} dx

and try the substitution u = (x-a)2 on the first one. Then you can use the fact that

\int_{-\infty}^\infty e^{-t^2}\, dt = \sqrt\pi
 
Ok. I gave it another shot but I don't seem to be able to come up with a reasonable solution.

<br /> \begin{equation}<br /> \frac{1}{\sqrt{4\pi Dt}}\int_{-\infty}^\infty x e^{-\frac{(x-t)^2}{4Dt}} dx=\frac{1}{\sqrt{4\pi Dt}} \left( \int_{-\infty}^\infty (x-dt) e^{-\frac{(x-dt)^2}{4Dt}} dx - dt \int_{-\infty}^\infty e^{-\frac{(x-dt)^2}{4Dt}} dx \right) =/u=(x-dt)/ = <br /> \end{equation}<br />
<br /> \begin{equation}<br /> \frac{1}{\sqrt{4\pi Dt}} \int_{-\infty}^\infty (u) e^{-\frac{u^2}{4Dt}} dx - \frac{1}{\sqrt{4\pi Dt}} dt \int_{-\infty}^\infty e^{-\frac{(x-dt)^2}{4Dt}} dx<br /> \end{equation}<br />

Now I'm pretty sure the right one becomes dt*1, but the other one has me a bit confused. Does the left one equal pi times the constant even though I have 4Dt in the denominator under u?
I'm just completely lost here.
 
Hint: The integrand of the left integral is an odd function of u. (And the dx should be du.)
 
andresthor said:
Ok. I gave it another shot but I don't seem to be able to come up with a reasonable solution.

<br /> \begin{equation}<br /> \frac{1}{\sqrt{4\pi Dt}}\int_{-\infty}^\infty x e^{-\frac{(x-t)^2}{4Dt}} dx=\frac{1}{\sqrt{4\pi Dt}} \left( \int_{-\infty}^\infty (x-dt) e^{-\frac{(x-dt)^2}{4Dt}} dx - dt \int_{-\infty}^\infty e^{-\frac{(x-dt)^2}{4Dt}} dx \right) =/u=(x-dt)/ = <br /> \end{equation}<br />
<br /> \begin{equation}<br /> \frac{1}{\sqrt{4\pi Dt}} \int_{-\infty}^\infty (u) e^{-\frac{u^2}{4Dt}} dx - \frac{1}{\sqrt{4\pi Dt}} dt \int_{-\infty}^\infty e^{-\frac{(x-dt)^2}{4Dt}} dx<br /> \end{equation}<br />


Now I'm pretty sure the right one becomes dt*1, but the other one has me a bit confused. Does the left one equal pi times the constant even though I have 4Dt in the denominator under u?
I'm just completely lost here.

In the integral on the left (which should have du instead of dx) try the substitution:

v = \frac {u^2} {\sqrt{4Dt}}

Or you could observe the integrand is an odd function :wink:
 
Of course! It's an odd function!

Thanks for the help, really appreciated.
 
Question: A clock's minute hand has length 4 and its hour hand has length 3. What is the distance between the tips at the moment when it is increasing most rapidly?(Putnam Exam Question) Answer: Making assumption that both the hands moves at constant angular velocities, the answer is ## \sqrt{7} .## But don't you think this assumption is somewhat doubtful and wrong?

Similar threads

Replies
1
Views
2K
Replies
6
Views
2K
  • · Replies 7 ·
Replies
7
Views
2K
Replies
3
Views
1K
  • · Replies 31 ·
2
Replies
31
Views
4K
  • · Replies 6 ·
Replies
6
Views
3K
  • · Replies 1 ·
Replies
1
Views
1K
Replies
2
Views
2K
  • · Replies 7 ·
Replies
7
Views
2K
Replies
1
Views
2K